Changing integration limits from [0,a] to [0, inf)

Click For Summary
Changing the limits of an integral from [0, a] to [0, ∞) while maintaining the same result is generally not feasible for an arbitrary function f(x). The integral over [0, a] depends solely on the values of f(x) within that interval, while the integral over [0, ∞) incorporates the behavior of f(u) for all positive values, including those beyond a. Although a substitution like u = tan(πx/2a) can transform the limits, it complicates the integral and does not yield a straightforward solution for finding a constant c. If both integrals are finite, c can be determined by dividing one by the other, but this scenario is limited. Ultimately, the relationship between the two integrals is not guaranteed without specific conditions on the function f.
PerUlven
Messages
7
Reaction score
0
Homework Statement
This isn't really homework, but a question I came upon when doing my homework.

How can I go from an integral with limits 0 and a:

<br /> \int_0^a f(x) dx<br />

to something with limits 0 and \infty (still giving the same answer)

<br /> c\int_0^\infty f(u) du<br />

, where c is some kind of constant.

The attempt at a solution
I've tried substituting x with tan(\frac{\pi x}{2a}), since this seems to give the correct limits, but I don't know if it makes any sense, and what to do next (how to find c for example).
 
Physics news on Phys.org
PerUlven said:
Homework Statement
This isn't really homework, but a question I came upon when doing my homework.

How can I go from an integral with limits 0 and a:

<br /> \int_0^a f(x) dx<br />

to something with limits 0 and \infty (still giving the same answer)

<br /> c\int_0^\infty f(u) du<br />

, where c is some kind of constant.

The attempt at a solution
I've tried substituting x with tan(\frac{\pi x}{2a}), since this seems to give the correct limits, but I don't know if it makes any sense, and what to do next (how to find c for example).

You can't, in general. The integral of f(x) for 0 ≤ x ≤ a depends only on the value of f(x) for x between 0 and a, while the integral of f(u) for 0 ≤ u < ∞ depends on the values of f(u) for all positive values of u, including values u > a. The behaviour of f on [0,a] may be essentially unrealated to the behaviour on [a,∞).

Now, of course, if both integrals are finite, you can get c by dividing one by the other, but that is trivial and probably not what you had in mind. You cannot do that at all if the integral over [0,a] is finite while that over [0,∞) is infinite.

RGV
 
I think it is not possible to change limits from some value to infinity for an unkown function... I can be wrong but I never saw any such thing before
 
PerUlven said:
Homework Statement
This isn't really homework, but a question I came upon when doing my homework.

How can I go from an integral with limits 0 and a:

<br /> \int_0^a f(x) dx<br />

to something with limits 0 and \infty (still giving the same answer)

<br /> c\int_0^\infty f(u) du<br />

, where c is some kind of constant.

The attempt at a solution
I've tried substituting x with \tan(\frac{\pi x}{2a}), since this seems to give the correct limits, but I don't know if it makes any sense, and what to do next (how to find c for example).

If you make the substitution, \displaystyle u=\tan\left(\frac{\pi x}{2a}\right)\,, then \displaystyle x=\frac{2a}{\pi }\tan^{-1}(u)\,, so that \displaystyle dx=\frac{2a}{\pi }\frac{1}{1+u^2}du\ .

This gives: \displaystyle \int_0^a f(x)\,dx<br /> =\frac{2a}{\pi }\int_0^{\infty} f\left(\frac{2a}{\pi }\tan^{-1}(u)\right)\frac{1}{1+u^2}du\,, which is probably not what you wanted.
 
Thanks a lot guys. This is what happens when I sit and wonder about mathematics and physics into the night!
 
Question: A clock's minute hand has length 4 and its hour hand has length 3. What is the distance between the tips at the moment when it is increasing most rapidly?(Putnam Exam Question) Answer: Making assumption that both the hands moves at constant angular velocities, the answer is ## \sqrt{7} .## But don't you think this assumption is somewhat doubtful and wrong?

Similar threads

Replies
2
Views
2K
  • · Replies 5 ·
Replies
5
Views
2K
  • · Replies 16 ·
Replies
16
Views
2K
  • · Replies 8 ·
Replies
8
Views
2K
  • · Replies 12 ·
Replies
12
Views
3K
  • · Replies 1 ·
Replies
1
Views
2K
  • · Replies 27 ·
Replies
27
Views
3K
  • · Replies 2 ·
Replies
2
Views
2K
  • · Replies 3 ·
Replies
3
Views
2K
  • · Replies 5 ·
Replies
5
Views
2K